Prove that $ X_a={x_a; x_a=(a,a^2,a^3…), a in R }$ are linearly independent.












1












$begingroup$


I need to prove that all vectors in the set $X_a={x_a; x_a=(a,a^2,a^3...), a in mathbb{R} } subset mathbb{R}^{infty} $are linearly independent



I have tryed to prove it by induction but I don't know how to formulate it. I have tryed also to prove it by using polynomials (I know how to show that they are linearly independent) but these are just scalars of a specific form not $(x,x^2,x^3...)$ where $x$ is a variable (and then so and so hold for every $x$)



any Ideas?










share|cite|improve this question











$endgroup$












  • $begingroup$
    In what vector space? $ell^2$? $R^infty$? Does that ellipsis mean it is infinite dimensional?
    $endgroup$
    – Jeffery Opoku-Mensah
    Oct 22 '18 at 21:53












  • $begingroup$
    so $x_a$ is a set?
    $endgroup$
    – mathworker21
    Oct 22 '18 at 21:53










  • $begingroup$
    $X_a$ is a set (the notation was a bit confusing so I changed it now) the vector space is $R^{infty}$
    $endgroup$
    – Alexandar Solženjicin
    Oct 22 '18 at 21:55


















1












$begingroup$


I need to prove that all vectors in the set $X_a={x_a; x_a=(a,a^2,a^3...), a in mathbb{R} } subset mathbb{R}^{infty} $are linearly independent



I have tryed to prove it by induction but I don't know how to formulate it. I have tryed also to prove it by using polynomials (I know how to show that they are linearly independent) but these are just scalars of a specific form not $(x,x^2,x^3...)$ where $x$ is a variable (and then so and so hold for every $x$)



any Ideas?










share|cite|improve this question











$endgroup$












  • $begingroup$
    In what vector space? $ell^2$? $R^infty$? Does that ellipsis mean it is infinite dimensional?
    $endgroup$
    – Jeffery Opoku-Mensah
    Oct 22 '18 at 21:53












  • $begingroup$
    so $x_a$ is a set?
    $endgroup$
    – mathworker21
    Oct 22 '18 at 21:53










  • $begingroup$
    $X_a$ is a set (the notation was a bit confusing so I changed it now) the vector space is $R^{infty}$
    $endgroup$
    – Alexandar Solženjicin
    Oct 22 '18 at 21:55
















1












1








1





$begingroup$


I need to prove that all vectors in the set $X_a={x_a; x_a=(a,a^2,a^3...), a in mathbb{R} } subset mathbb{R}^{infty} $are linearly independent



I have tryed to prove it by induction but I don't know how to formulate it. I have tryed also to prove it by using polynomials (I know how to show that they are linearly independent) but these are just scalars of a specific form not $(x,x^2,x^3...)$ where $x$ is a variable (and then so and so hold for every $x$)



any Ideas?










share|cite|improve this question











$endgroup$




I need to prove that all vectors in the set $X_a={x_a; x_a=(a,a^2,a^3...), a in mathbb{R} } subset mathbb{R}^{infty} $are linearly independent



I have tryed to prove it by induction but I don't know how to formulate it. I have tryed also to prove it by using polynomials (I know how to show that they are linearly independent) but these are just scalars of a specific form not $(x,x^2,x^3...)$ where $x$ is a variable (and then so and so hold for every $x$)



any Ideas?







linear-algebra polynomials vector-spaces






share|cite|improve this question















share|cite|improve this question













share|cite|improve this question




share|cite|improve this question








edited Dec 6 '18 at 12:17







Alexandar Solženjicin

















asked Oct 22 '18 at 21:48









Alexandar SolženjicinAlexandar Solženjicin

858




858












  • $begingroup$
    In what vector space? $ell^2$? $R^infty$? Does that ellipsis mean it is infinite dimensional?
    $endgroup$
    – Jeffery Opoku-Mensah
    Oct 22 '18 at 21:53












  • $begingroup$
    so $x_a$ is a set?
    $endgroup$
    – mathworker21
    Oct 22 '18 at 21:53










  • $begingroup$
    $X_a$ is a set (the notation was a bit confusing so I changed it now) the vector space is $R^{infty}$
    $endgroup$
    – Alexandar Solženjicin
    Oct 22 '18 at 21:55




















  • $begingroup$
    In what vector space? $ell^2$? $R^infty$? Does that ellipsis mean it is infinite dimensional?
    $endgroup$
    – Jeffery Opoku-Mensah
    Oct 22 '18 at 21:53












  • $begingroup$
    so $x_a$ is a set?
    $endgroup$
    – mathworker21
    Oct 22 '18 at 21:53










  • $begingroup$
    $X_a$ is a set (the notation was a bit confusing so I changed it now) the vector space is $R^{infty}$
    $endgroup$
    – Alexandar Solženjicin
    Oct 22 '18 at 21:55


















$begingroup$
In what vector space? $ell^2$? $R^infty$? Does that ellipsis mean it is infinite dimensional?
$endgroup$
– Jeffery Opoku-Mensah
Oct 22 '18 at 21:53






$begingroup$
In what vector space? $ell^2$? $R^infty$? Does that ellipsis mean it is infinite dimensional?
$endgroup$
– Jeffery Opoku-Mensah
Oct 22 '18 at 21:53














$begingroup$
so $x_a$ is a set?
$endgroup$
– mathworker21
Oct 22 '18 at 21:53




$begingroup$
so $x_a$ is a set?
$endgroup$
– mathworker21
Oct 22 '18 at 21:53












$begingroup$
$X_a$ is a set (the notation was a bit confusing so I changed it now) the vector space is $R^{infty}$
$endgroup$
– Alexandar Solženjicin
Oct 22 '18 at 21:55






$begingroup$
$X_a$ is a set (the notation was a bit confusing so I changed it now) the vector space is $R^{infty}$
$endgroup$
– Alexandar Solženjicin
Oct 22 '18 at 21:55












2 Answers
2






active

oldest

votes


















3












$begingroup$

I assume you mean $ane 0$, otherwise it is of course not true. Well, let $a_1,a_2,...,a_n$ be different non zero real numbers and write a linear combination $lambda_1x_{a_1}+lambda_2x_{a_2}+...+lambda_nx_{a_n}=0$. Here the zero vector is $x_0$, all its coordinates are zeros, including the first $n$ coordinates of course. So we get a system of linear equations:



$lambda_1a_1+lambda_2a_2+...+lambda_na_n=0$



$lambda_1a_1^2+lambda_2a_2^2+...+lambda_na_n^2=0$



.................................................



$lambda_1a_1^n+lambda_2a_n^2+...+lambda_na_n^n=0$



This is a system of linear equations with $lambda_1,...,lambda_n$ being the variables. To show $lambda_1=...=lambda_n=0$ it is enough to prove that the determinant of the matrix which represents the system is not zero. Write the matrix and see that its determinant is $a_1a_2...a_n$ times the determinant of a Vandermonde matrix which I assume you know is not zero. (of course we are using the fact that $a_1,a_2,...,a_n$ are all different)






share|cite|improve this answer









$endgroup$





















    1












    $begingroup$

    Assume that there exist a nonempty minimally sized set of nonzero elements



    $$mathcal{S} = left{(x_0, x_0^2, x_0^3ldots), ldots , (x_n, x_n^2, x_n^3 ldots)right}
    $$



    that are linearly dependent. Then



    $$x_0^k = sum_{i=1}^{n} c_i x_i^k$$



    for all $k in mathbb{N}_0$. Then



    $$sum_{i=1}^{n} (c_ix_i) x_i^{k-1} = x_0^k = x_0(x_0)^{k-1} = sum_{i=1}^{n} (x_0c_i) x_i^{k-1},$$



    which implies that for all $k in mathbb{N}$,



    $$sum_{i=1}^{n} (x_0c_i - x_ic_i)x_i^{k-1} = 0$$



    Since $mathcal{S} setminus {(x_0, x_0^2, x_0^3, ldots )}$ is by assumption linearly independent, it must be that for each $i$, $x_0c_i - x_ic_i = 0$, thus $x_0 = x_i$, or $c_i = 0$. In other words, either
    $$x_0 = x_1 = ldots = x_n,$$
    or $mathbf{0} in mathcal{S}$.
    Contradiction.






    share|cite|improve this answer









    $endgroup$













    • $begingroup$
      is $x_0^k=(x_0,x_0^2..) $or je k-th element of that list, or something else?
      $endgroup$
      – Alexandar Solženjicin
      Oct 22 '18 at 22:17












    • $begingroup$
      $x_0$ is just the first element of the vector $(x_0, x_0^2, x_0^3, ldots)$. I didn't use your notation since it would be a bit clunky. Then $x_0^k$ is that raised to the $kth$ power.
      $endgroup$
      – Jeffery Opoku-Mensah
      Oct 22 '18 at 22:18












    • $begingroup$
      what if $S$ is not minimaly sized? and then $ S/{(x_0,x_0^2..)}$ is not linearly independent?
      $endgroup$
      – Alexandar Solženjicin
      Oct 22 '18 at 22:37












    • $begingroup$
      If there exists a linearly dependent set of vectors, then there must be a finite minimally sized set of linearly dependent vectors. Thus if there is no minimally sized linearly dependent set of vectors, there is no linearly dependent set of vectors at all.
      $endgroup$
      – Jeffery Opoku-Mensah
      Oct 22 '18 at 22:55













    Your Answer





    StackExchange.ifUsing("editor", function () {
    return StackExchange.using("mathjaxEditing", function () {
    StackExchange.MarkdownEditor.creationCallbacks.add(function (editor, postfix) {
    StackExchange.mathjaxEditing.prepareWmdForMathJax(editor, postfix, [["$", "$"], ["\\(","\\)"]]);
    });
    });
    }, "mathjax-editing");

    StackExchange.ready(function() {
    var channelOptions = {
    tags: "".split(" "),
    id: "69"
    };
    initTagRenderer("".split(" "), "".split(" "), channelOptions);

    StackExchange.using("externalEditor", function() {
    // Have to fire editor after snippets, if snippets enabled
    if (StackExchange.settings.snippets.snippetsEnabled) {
    StackExchange.using("snippets", function() {
    createEditor();
    });
    }
    else {
    createEditor();
    }
    });

    function createEditor() {
    StackExchange.prepareEditor({
    heartbeatType: 'answer',
    autoActivateHeartbeat: false,
    convertImagesToLinks: true,
    noModals: true,
    showLowRepImageUploadWarning: true,
    reputationToPostImages: 10,
    bindNavPrevention: true,
    postfix: "",
    imageUploader: {
    brandingHtml: "Powered by u003ca class="icon-imgur-white" href="https://imgur.com/"u003eu003c/au003e",
    contentPolicyHtml: "User contributions licensed under u003ca href="https://creativecommons.org/licenses/by-sa/3.0/"u003ecc by-sa 3.0 with attribution requiredu003c/au003e u003ca href="https://stackoverflow.com/legal/content-policy"u003e(content policy)u003c/au003e",
    allowUrls: true
    },
    noCode: true, onDemand: true,
    discardSelector: ".discard-answer"
    ,immediatelyShowMarkdownHelp:true
    });


    }
    });














    draft saved

    draft discarded


















    StackExchange.ready(
    function () {
    StackExchange.openid.initPostLogin('.new-post-login', 'https%3a%2f%2fmath.stackexchange.com%2fquestions%2f2966716%2fprove-that-x-a-x-a-x-a-a-a2-a3-a-in-r-are-linearly-independent%23new-answer', 'question_page');
    }
    );

    Post as a guest















    Required, but never shown

























    2 Answers
    2






    active

    oldest

    votes








    2 Answers
    2






    active

    oldest

    votes









    active

    oldest

    votes






    active

    oldest

    votes









    3












    $begingroup$

    I assume you mean $ane 0$, otherwise it is of course not true. Well, let $a_1,a_2,...,a_n$ be different non zero real numbers and write a linear combination $lambda_1x_{a_1}+lambda_2x_{a_2}+...+lambda_nx_{a_n}=0$. Here the zero vector is $x_0$, all its coordinates are zeros, including the first $n$ coordinates of course. So we get a system of linear equations:



    $lambda_1a_1+lambda_2a_2+...+lambda_na_n=0$



    $lambda_1a_1^2+lambda_2a_2^2+...+lambda_na_n^2=0$



    .................................................



    $lambda_1a_1^n+lambda_2a_n^2+...+lambda_na_n^n=0$



    This is a system of linear equations with $lambda_1,...,lambda_n$ being the variables. To show $lambda_1=...=lambda_n=0$ it is enough to prove that the determinant of the matrix which represents the system is not zero. Write the matrix and see that its determinant is $a_1a_2...a_n$ times the determinant of a Vandermonde matrix which I assume you know is not zero. (of course we are using the fact that $a_1,a_2,...,a_n$ are all different)






    share|cite|improve this answer









    $endgroup$


















      3












      $begingroup$

      I assume you mean $ane 0$, otherwise it is of course not true. Well, let $a_1,a_2,...,a_n$ be different non zero real numbers and write a linear combination $lambda_1x_{a_1}+lambda_2x_{a_2}+...+lambda_nx_{a_n}=0$. Here the zero vector is $x_0$, all its coordinates are zeros, including the first $n$ coordinates of course. So we get a system of linear equations:



      $lambda_1a_1+lambda_2a_2+...+lambda_na_n=0$



      $lambda_1a_1^2+lambda_2a_2^2+...+lambda_na_n^2=0$



      .................................................



      $lambda_1a_1^n+lambda_2a_n^2+...+lambda_na_n^n=0$



      This is a system of linear equations with $lambda_1,...,lambda_n$ being the variables. To show $lambda_1=...=lambda_n=0$ it is enough to prove that the determinant of the matrix which represents the system is not zero. Write the matrix and see that its determinant is $a_1a_2...a_n$ times the determinant of a Vandermonde matrix which I assume you know is not zero. (of course we are using the fact that $a_1,a_2,...,a_n$ are all different)






      share|cite|improve this answer









      $endgroup$
















        3












        3








        3





        $begingroup$

        I assume you mean $ane 0$, otherwise it is of course not true. Well, let $a_1,a_2,...,a_n$ be different non zero real numbers and write a linear combination $lambda_1x_{a_1}+lambda_2x_{a_2}+...+lambda_nx_{a_n}=0$. Here the zero vector is $x_0$, all its coordinates are zeros, including the first $n$ coordinates of course. So we get a system of linear equations:



        $lambda_1a_1+lambda_2a_2+...+lambda_na_n=0$



        $lambda_1a_1^2+lambda_2a_2^2+...+lambda_na_n^2=0$



        .................................................



        $lambda_1a_1^n+lambda_2a_n^2+...+lambda_na_n^n=0$



        This is a system of linear equations with $lambda_1,...,lambda_n$ being the variables. To show $lambda_1=...=lambda_n=0$ it is enough to prove that the determinant of the matrix which represents the system is not zero. Write the matrix and see that its determinant is $a_1a_2...a_n$ times the determinant of a Vandermonde matrix which I assume you know is not zero. (of course we are using the fact that $a_1,a_2,...,a_n$ are all different)






        share|cite|improve this answer









        $endgroup$



        I assume you mean $ane 0$, otherwise it is of course not true. Well, let $a_1,a_2,...,a_n$ be different non zero real numbers and write a linear combination $lambda_1x_{a_1}+lambda_2x_{a_2}+...+lambda_nx_{a_n}=0$. Here the zero vector is $x_0$, all its coordinates are zeros, including the first $n$ coordinates of course. So we get a system of linear equations:



        $lambda_1a_1+lambda_2a_2+...+lambda_na_n=0$



        $lambda_1a_1^2+lambda_2a_2^2+...+lambda_na_n^2=0$



        .................................................



        $lambda_1a_1^n+lambda_2a_n^2+...+lambda_na_n^n=0$



        This is a system of linear equations with $lambda_1,...,lambda_n$ being the variables. To show $lambda_1=...=lambda_n=0$ it is enough to prove that the determinant of the matrix which represents the system is not zero. Write the matrix and see that its determinant is $a_1a_2...a_n$ times the determinant of a Vandermonde matrix which I assume you know is not zero. (of course we are using the fact that $a_1,a_2,...,a_n$ are all different)







        share|cite|improve this answer












        share|cite|improve this answer



        share|cite|improve this answer










        answered Oct 22 '18 at 22:03









        MarkMark

        6,488416




        6,488416























            1












            $begingroup$

            Assume that there exist a nonempty minimally sized set of nonzero elements



            $$mathcal{S} = left{(x_0, x_0^2, x_0^3ldots), ldots , (x_n, x_n^2, x_n^3 ldots)right}
            $$



            that are linearly dependent. Then



            $$x_0^k = sum_{i=1}^{n} c_i x_i^k$$



            for all $k in mathbb{N}_0$. Then



            $$sum_{i=1}^{n} (c_ix_i) x_i^{k-1} = x_0^k = x_0(x_0)^{k-1} = sum_{i=1}^{n} (x_0c_i) x_i^{k-1},$$



            which implies that for all $k in mathbb{N}$,



            $$sum_{i=1}^{n} (x_0c_i - x_ic_i)x_i^{k-1} = 0$$



            Since $mathcal{S} setminus {(x_0, x_0^2, x_0^3, ldots )}$ is by assumption linearly independent, it must be that for each $i$, $x_0c_i - x_ic_i = 0$, thus $x_0 = x_i$, or $c_i = 0$. In other words, either
            $$x_0 = x_1 = ldots = x_n,$$
            or $mathbf{0} in mathcal{S}$.
            Contradiction.






            share|cite|improve this answer









            $endgroup$













            • $begingroup$
              is $x_0^k=(x_0,x_0^2..) $or je k-th element of that list, or something else?
              $endgroup$
              – Alexandar Solženjicin
              Oct 22 '18 at 22:17












            • $begingroup$
              $x_0$ is just the first element of the vector $(x_0, x_0^2, x_0^3, ldots)$. I didn't use your notation since it would be a bit clunky. Then $x_0^k$ is that raised to the $kth$ power.
              $endgroup$
              – Jeffery Opoku-Mensah
              Oct 22 '18 at 22:18












            • $begingroup$
              what if $S$ is not minimaly sized? and then $ S/{(x_0,x_0^2..)}$ is not linearly independent?
              $endgroup$
              – Alexandar Solženjicin
              Oct 22 '18 at 22:37












            • $begingroup$
              If there exists a linearly dependent set of vectors, then there must be a finite minimally sized set of linearly dependent vectors. Thus if there is no minimally sized linearly dependent set of vectors, there is no linearly dependent set of vectors at all.
              $endgroup$
              – Jeffery Opoku-Mensah
              Oct 22 '18 at 22:55


















            1












            $begingroup$

            Assume that there exist a nonempty minimally sized set of nonzero elements



            $$mathcal{S} = left{(x_0, x_0^2, x_0^3ldots), ldots , (x_n, x_n^2, x_n^3 ldots)right}
            $$



            that are linearly dependent. Then



            $$x_0^k = sum_{i=1}^{n} c_i x_i^k$$



            for all $k in mathbb{N}_0$. Then



            $$sum_{i=1}^{n} (c_ix_i) x_i^{k-1} = x_0^k = x_0(x_0)^{k-1} = sum_{i=1}^{n} (x_0c_i) x_i^{k-1},$$



            which implies that for all $k in mathbb{N}$,



            $$sum_{i=1}^{n} (x_0c_i - x_ic_i)x_i^{k-1} = 0$$



            Since $mathcal{S} setminus {(x_0, x_0^2, x_0^3, ldots )}$ is by assumption linearly independent, it must be that for each $i$, $x_0c_i - x_ic_i = 0$, thus $x_0 = x_i$, or $c_i = 0$. In other words, either
            $$x_0 = x_1 = ldots = x_n,$$
            or $mathbf{0} in mathcal{S}$.
            Contradiction.






            share|cite|improve this answer









            $endgroup$













            • $begingroup$
              is $x_0^k=(x_0,x_0^2..) $or je k-th element of that list, or something else?
              $endgroup$
              – Alexandar Solženjicin
              Oct 22 '18 at 22:17












            • $begingroup$
              $x_0$ is just the first element of the vector $(x_0, x_0^2, x_0^3, ldots)$. I didn't use your notation since it would be a bit clunky. Then $x_0^k$ is that raised to the $kth$ power.
              $endgroup$
              – Jeffery Opoku-Mensah
              Oct 22 '18 at 22:18












            • $begingroup$
              what if $S$ is not minimaly sized? and then $ S/{(x_0,x_0^2..)}$ is not linearly independent?
              $endgroup$
              – Alexandar Solženjicin
              Oct 22 '18 at 22:37












            • $begingroup$
              If there exists a linearly dependent set of vectors, then there must be a finite minimally sized set of linearly dependent vectors. Thus if there is no minimally sized linearly dependent set of vectors, there is no linearly dependent set of vectors at all.
              $endgroup$
              – Jeffery Opoku-Mensah
              Oct 22 '18 at 22:55
















            1












            1








            1





            $begingroup$

            Assume that there exist a nonempty minimally sized set of nonzero elements



            $$mathcal{S} = left{(x_0, x_0^2, x_0^3ldots), ldots , (x_n, x_n^2, x_n^3 ldots)right}
            $$



            that are linearly dependent. Then



            $$x_0^k = sum_{i=1}^{n} c_i x_i^k$$



            for all $k in mathbb{N}_0$. Then



            $$sum_{i=1}^{n} (c_ix_i) x_i^{k-1} = x_0^k = x_0(x_0)^{k-1} = sum_{i=1}^{n} (x_0c_i) x_i^{k-1},$$



            which implies that for all $k in mathbb{N}$,



            $$sum_{i=1}^{n} (x_0c_i - x_ic_i)x_i^{k-1} = 0$$



            Since $mathcal{S} setminus {(x_0, x_0^2, x_0^3, ldots )}$ is by assumption linearly independent, it must be that for each $i$, $x_0c_i - x_ic_i = 0$, thus $x_0 = x_i$, or $c_i = 0$. In other words, either
            $$x_0 = x_1 = ldots = x_n,$$
            or $mathbf{0} in mathcal{S}$.
            Contradiction.






            share|cite|improve this answer









            $endgroup$



            Assume that there exist a nonempty minimally sized set of nonzero elements



            $$mathcal{S} = left{(x_0, x_0^2, x_0^3ldots), ldots , (x_n, x_n^2, x_n^3 ldots)right}
            $$



            that are linearly dependent. Then



            $$x_0^k = sum_{i=1}^{n} c_i x_i^k$$



            for all $k in mathbb{N}_0$. Then



            $$sum_{i=1}^{n} (c_ix_i) x_i^{k-1} = x_0^k = x_0(x_0)^{k-1} = sum_{i=1}^{n} (x_0c_i) x_i^{k-1},$$



            which implies that for all $k in mathbb{N}$,



            $$sum_{i=1}^{n} (x_0c_i - x_ic_i)x_i^{k-1} = 0$$



            Since $mathcal{S} setminus {(x_0, x_0^2, x_0^3, ldots )}$ is by assumption linearly independent, it must be that for each $i$, $x_0c_i - x_ic_i = 0$, thus $x_0 = x_i$, or $c_i = 0$. In other words, either
            $$x_0 = x_1 = ldots = x_n,$$
            or $mathbf{0} in mathcal{S}$.
            Contradiction.







            share|cite|improve this answer












            share|cite|improve this answer



            share|cite|improve this answer










            answered Oct 22 '18 at 22:14









            Jeffery Opoku-MensahJeffery Opoku-Mensah

            3,528720




            3,528720












            • $begingroup$
              is $x_0^k=(x_0,x_0^2..) $or je k-th element of that list, or something else?
              $endgroup$
              – Alexandar Solženjicin
              Oct 22 '18 at 22:17












            • $begingroup$
              $x_0$ is just the first element of the vector $(x_0, x_0^2, x_0^3, ldots)$. I didn't use your notation since it would be a bit clunky. Then $x_0^k$ is that raised to the $kth$ power.
              $endgroup$
              – Jeffery Opoku-Mensah
              Oct 22 '18 at 22:18












            • $begingroup$
              what if $S$ is not minimaly sized? and then $ S/{(x_0,x_0^2..)}$ is not linearly independent?
              $endgroup$
              – Alexandar Solženjicin
              Oct 22 '18 at 22:37












            • $begingroup$
              If there exists a linearly dependent set of vectors, then there must be a finite minimally sized set of linearly dependent vectors. Thus if there is no minimally sized linearly dependent set of vectors, there is no linearly dependent set of vectors at all.
              $endgroup$
              – Jeffery Opoku-Mensah
              Oct 22 '18 at 22:55




















            • $begingroup$
              is $x_0^k=(x_0,x_0^2..) $or je k-th element of that list, or something else?
              $endgroup$
              – Alexandar Solženjicin
              Oct 22 '18 at 22:17












            • $begingroup$
              $x_0$ is just the first element of the vector $(x_0, x_0^2, x_0^3, ldots)$. I didn't use your notation since it would be a bit clunky. Then $x_0^k$ is that raised to the $kth$ power.
              $endgroup$
              – Jeffery Opoku-Mensah
              Oct 22 '18 at 22:18












            • $begingroup$
              what if $S$ is not minimaly sized? and then $ S/{(x_0,x_0^2..)}$ is not linearly independent?
              $endgroup$
              – Alexandar Solženjicin
              Oct 22 '18 at 22:37












            • $begingroup$
              If there exists a linearly dependent set of vectors, then there must be a finite minimally sized set of linearly dependent vectors. Thus if there is no minimally sized linearly dependent set of vectors, there is no linearly dependent set of vectors at all.
              $endgroup$
              – Jeffery Opoku-Mensah
              Oct 22 '18 at 22:55


















            $begingroup$
            is $x_0^k=(x_0,x_0^2..) $or je k-th element of that list, or something else?
            $endgroup$
            – Alexandar Solženjicin
            Oct 22 '18 at 22:17






            $begingroup$
            is $x_0^k=(x_0,x_0^2..) $or je k-th element of that list, or something else?
            $endgroup$
            – Alexandar Solženjicin
            Oct 22 '18 at 22:17














            $begingroup$
            $x_0$ is just the first element of the vector $(x_0, x_0^2, x_0^3, ldots)$. I didn't use your notation since it would be a bit clunky. Then $x_0^k$ is that raised to the $kth$ power.
            $endgroup$
            – Jeffery Opoku-Mensah
            Oct 22 '18 at 22:18






            $begingroup$
            $x_0$ is just the first element of the vector $(x_0, x_0^2, x_0^3, ldots)$. I didn't use your notation since it would be a bit clunky. Then $x_0^k$ is that raised to the $kth$ power.
            $endgroup$
            – Jeffery Opoku-Mensah
            Oct 22 '18 at 22:18














            $begingroup$
            what if $S$ is not minimaly sized? and then $ S/{(x_0,x_0^2..)}$ is not linearly independent?
            $endgroup$
            – Alexandar Solženjicin
            Oct 22 '18 at 22:37






            $begingroup$
            what if $S$ is not minimaly sized? and then $ S/{(x_0,x_0^2..)}$ is not linearly independent?
            $endgroup$
            – Alexandar Solženjicin
            Oct 22 '18 at 22:37














            $begingroup$
            If there exists a linearly dependent set of vectors, then there must be a finite minimally sized set of linearly dependent vectors. Thus if there is no minimally sized linearly dependent set of vectors, there is no linearly dependent set of vectors at all.
            $endgroup$
            – Jeffery Opoku-Mensah
            Oct 22 '18 at 22:55






            $begingroup$
            If there exists a linearly dependent set of vectors, then there must be a finite minimally sized set of linearly dependent vectors. Thus if there is no minimally sized linearly dependent set of vectors, there is no linearly dependent set of vectors at all.
            $endgroup$
            – Jeffery Opoku-Mensah
            Oct 22 '18 at 22:55




















            draft saved

            draft discarded




















































            Thanks for contributing an answer to Mathematics Stack Exchange!


            • Please be sure to answer the question. Provide details and share your research!

            But avoid



            • Asking for help, clarification, or responding to other answers.

            • Making statements based on opinion; back them up with references or personal experience.


            Use MathJax to format equations. MathJax reference.


            To learn more, see our tips on writing great answers.




            draft saved


            draft discarded














            StackExchange.ready(
            function () {
            StackExchange.openid.initPostLogin('.new-post-login', 'https%3a%2f%2fmath.stackexchange.com%2fquestions%2f2966716%2fprove-that-x-a-x-a-x-a-a-a2-a3-a-in-r-are-linearly-independent%23new-answer', 'question_page');
            }
            );

            Post as a guest















            Required, but never shown





















































            Required, but never shown














            Required, but never shown












            Required, but never shown







            Required, but never shown

































            Required, but never shown














            Required, but never shown












            Required, but never shown







            Required, but never shown







            Popular posts from this blog

            To store a contact into the json file from server.js file using a class in NodeJS

            Redirect URL with Chrome Remote Debugging Android Devices

            Dieringhausen